LSAT and Law School Admissions Forum

Get expert LSAT preparation and law school admissions advice from PowerScore Test Preparation.

 Administrator
PowerScore Staff
  • PowerScore Staff
  • Posts: 8919
  • Joined: Feb 02, 2011
|
#33166
Complete Question Explanation
(See the complete passage discussion here: lsat/viewtopic.php?t=13636)

The correct answer choice is (B)

The answer to this Cannot Be True, Author’s Perspective question will be the choice with which, based on the statements from the stimulus, the author clearly would not agree. The four incorrect answer choices will be the ones that either the author would agree with, or points with which the author would not necessarily disagree.

Answer choice (A): This answer choice dealing with certain abnormally shaped prions is confirmed by the passage to be true, with regard to CJD in particular. Since the author would agree with this choice based on the information in the passage, it cannot be the right answer to this Cannot Be True question.

Answer choice (B): This is the correct answer choice. On line 45, the author specifies that CJD is inevitably fatal. Thus, the author would clearly disagree with the statement that some of those afflicted with the condition eventually recover, confirming this as the right answer to this Cannot Be True question.

Answer choice (C): The author points out in the second paragraph that the main point of distinction between the prion and the four previously known pathogenic agents is that prions are not made of the same genetic material as the others, so this choice should be ruled out of contention in response to this question.

Answer choice (D): In the third paragraph, the passage confirms this answer choice, saying that the lack of response from the immune system makes CJD eventually fatal.

Answer choice (E): The author closes the last paragraph saying that prions’ exact mechanisms are not yet understood. Since the author would clearly agree with this choice, it should be ruled out of contention for this Cannot Be True question.
 soobin903
  • Posts: 4
  • Joined: Nov 13, 2019
|
#74938
Hi, I have a simple(?) question regarding whether the answer choice (B) could be inferred from the passage.

In lines 42 through 45, it says that "in the absence of any effective therapy... CJD is inevitably fatal."
I've understood the sentence to mean that CJD would not be necessarily fatal if there is an effective therapy, and since there is no evidence in the given passage that there is NO effective therapy, I thought perhaps the answer choice (B) "some will recover" could be inferred... especially on the grounds that the progress of the disease varies from person to person.

Am I stretching too far out of the passage's scope, because of the word "WILL" in the answer choice? In other words, could B be inferred if B was mitigated to "Some patients infected with CJD MIGHT recover from the disease", or would it still need the specific condition of an effective therapy (B would be right only if the condition is included as such : some patients infected with CJD MIGHT recover from the disease IF AN EFFECTIVE THERAPY IS DEVELOPED) in order for B to be considered as an answer choice with which the author would agree?

Thank you in advance! All of your explanations really help a lot :)
 Jeremy Press
PowerScore Staff
  • PowerScore Staff
  • Posts: 1000
  • Joined: Jun 12, 2017
|
#74958
Hi soobin,

That's a really great observation, and I agree with you that absent some evidence that the author believes there isn't currently an effective therapy, there is some possibility that the author would believe some people will recover from CJD. However, there is one fairly easy to miss, and perhaps not completely dispositive but at the very least suggestive, clue in the passage that the author believes an effective therapy probably does not currently exist. In the last sentence of the passage, the author tells us that "the exact mechanisms by which prions reproduce themselves and cause cellular destruction have yet to be completely understood," which means it's relatively unlikely that we could identify an effective therapy. In other words, to stop the destruction prions cause, we probably need to learn first how they reproduce and cause such destruction (and then target our therapy to those mechanisms). Is it theoretically possible that we could stumble on an effective therapy before we had such mechanistic knowledge? I suppose, but that seems relatively unlikely, and makes answer choice B one that (even if it does not thoroughly logically contradict the author's stance) the author is least likely to agree with.

I hope this helps!

Jeremy
 soobin903
  • Posts: 4
  • Joined: Nov 13, 2019
|
#74989
Yes, that helps a lot! Thank you for the explanation :)

Get the most out of your LSAT Prep Plus subscription.

Analyze and track your performance with our Testing and Analytics Package.